An equal expression to $sum^{z}_{n=0}nfrac{x^{z-n}}{(z-n)!}$












0












$begingroup$


Is there a way to rewrite an equal expression to the following



$$sum^{z}_{n=0}nfrac{x^{z-n}}{(z-n)!}$$



but without the sum?



$z$ is integer and $xgeq 0$.










share|cite|improve this question









$endgroup$

















    0












    $begingroup$


    Is there a way to rewrite an equal expression to the following



    $$sum^{z}_{n=0}nfrac{x^{z-n}}{(z-n)!}$$



    but without the sum?



    $z$ is integer and $xgeq 0$.










    share|cite|improve this question









    $endgroup$















      0












      0








      0





      $begingroup$


      Is there a way to rewrite an equal expression to the following



      $$sum^{z}_{n=0}nfrac{x^{z-n}}{(z-n)!}$$



      but without the sum?



      $z$ is integer and $xgeq 0$.










      share|cite|improve this question









      $endgroup$




      Is there a way to rewrite an equal expression to the following



      $$sum^{z}_{n=0}nfrac{x^{z-n}}{(z-n)!}$$



      but without the sum?



      $z$ is integer and $xgeq 0$.







      real-analysis calculus sequences-and-series summation






      share|cite|improve this question













      share|cite|improve this question











      share|cite|improve this question




      share|cite|improve this question










      asked Jan 3 at 22:31









      Y.LY.L

      597




      597






















          2 Answers
          2






          active

          oldest

          votes


















          0












          $begingroup$

          The answer is no. By making the change of avriable $k=z-n$ and splitting the sum into two parts you get $(z-x)sum_{k=1}^{z-1} frac {x^{k}} {k!} +frac {zx^{z}} {z!}$ from which you cannot get rid of the sum.






          share|cite|improve this answer









          $endgroup$













          • $begingroup$
            Thank you @Kavi Rama Murthy, but you gave the answer. I just use the Gamma function for the last sum and I get the expression.
            $endgroup$
            – Y.L
            Jan 4 at 0:28



















          0












          $begingroup$

          Sticking to a more standard notation ($z$ normally denotes complex, not integral, number)



          begin{align}
          sum_{k=0}^n k frac{x^{n-k}}{(n-k)!}
          &= sum_{k=0}^{n} (n-k) frac{x^k}{k!}
          = nsum_{k=0}^n frac{x^k}{k!} - sum_{k=0}^n xfrac{d}{dx} frac{x^k}{k!}\
          &= nP_n(x) - xfrac{d}{dx}P_n(x)
          end{align}



          where $P_n(x) = sum_{k=0}^n frac{x^k}{k!}$ is the partial sum of the exponential's Taylor series. But as can be seen here, the incomplete gamma function $Gamma(n,x) = int_x^{infty} x^{n-1}e^{-x};dx$ may be expressed as



          $$Gamma(n,x) = (n-1)! e^{-x} sum_{k=0}^{n-1}frac{x^k}{k!}$$
          hence,
          $$P_n(x) = frac{Gamma(n+1,x)}{n!} e^x$$
          We may then compute



          begin{align}
          frac{d}{dx}P_n(x)
          &= P_n(x) frac{d}{dx} ln P_n(x) = P_n(x) left[,frac{frac{partial}{partial{x}}Gamma(n+1,x)}{Gamma(n+1,x)} + 1,right]\
          &= frac{Gamma(n+1,x)}{n!} e^x left[,frac{-x^ne^{-x}}{Gamma(n+1,x)} + 1,right]\
          &= frac{Gamma(n+1,x)}{n!} e^x - frac{x^n}{n!}
          end{align}



          Solving and simplifying, we get



          begin{align}
          sum_{k=0}^n k frac{x^{n-k}}{(n-k)!}
          &= nP_n(x) - xfrac{d}{dx}P_n(x)\
          &= frac{1}{n!} left[,(n-x)Gamma(n+1,x)e^x + x^{n+1},right]
          end{align}






          share|cite|improve this answer









          $endgroup$













            Your Answer





            StackExchange.ifUsing("editor", function () {
            return StackExchange.using("mathjaxEditing", function () {
            StackExchange.MarkdownEditor.creationCallbacks.add(function (editor, postfix) {
            StackExchange.mathjaxEditing.prepareWmdForMathJax(editor, postfix, [["$", "$"], ["\\(","\\)"]]);
            });
            });
            }, "mathjax-editing");

            StackExchange.ready(function() {
            var channelOptions = {
            tags: "".split(" "),
            id: "69"
            };
            initTagRenderer("".split(" "), "".split(" "), channelOptions);

            StackExchange.using("externalEditor", function() {
            // Have to fire editor after snippets, if snippets enabled
            if (StackExchange.settings.snippets.snippetsEnabled) {
            StackExchange.using("snippets", function() {
            createEditor();
            });
            }
            else {
            createEditor();
            }
            });

            function createEditor() {
            StackExchange.prepareEditor({
            heartbeatType: 'answer',
            autoActivateHeartbeat: false,
            convertImagesToLinks: true,
            noModals: true,
            showLowRepImageUploadWarning: true,
            reputationToPostImages: 10,
            bindNavPrevention: true,
            postfix: "",
            imageUploader: {
            brandingHtml: "Powered by u003ca class="icon-imgur-white" href="https://imgur.com/"u003eu003c/au003e",
            contentPolicyHtml: "User contributions licensed under u003ca href="https://creativecommons.org/licenses/by-sa/3.0/"u003ecc by-sa 3.0 with attribution requiredu003c/au003e u003ca href="https://stackoverflow.com/legal/content-policy"u003e(content policy)u003c/au003e",
            allowUrls: true
            },
            noCode: true, onDemand: true,
            discardSelector: ".discard-answer"
            ,immediatelyShowMarkdownHelp:true
            });


            }
            });














            draft saved

            draft discarded


















            StackExchange.ready(
            function () {
            StackExchange.openid.initPostLogin('.new-post-login', 'https%3a%2f%2fmath.stackexchange.com%2fquestions%2f3061092%2fan-equal-expression-to-sumz-n-0n-fracxz-nz-n%23new-answer', 'question_page');
            }
            );

            Post as a guest















            Required, but never shown

























            2 Answers
            2






            active

            oldest

            votes








            2 Answers
            2






            active

            oldest

            votes









            active

            oldest

            votes






            active

            oldest

            votes









            0












            $begingroup$

            The answer is no. By making the change of avriable $k=z-n$ and splitting the sum into two parts you get $(z-x)sum_{k=1}^{z-1} frac {x^{k}} {k!} +frac {zx^{z}} {z!}$ from which you cannot get rid of the sum.






            share|cite|improve this answer









            $endgroup$













            • $begingroup$
              Thank you @Kavi Rama Murthy, but you gave the answer. I just use the Gamma function for the last sum and I get the expression.
              $endgroup$
              – Y.L
              Jan 4 at 0:28
















            0












            $begingroup$

            The answer is no. By making the change of avriable $k=z-n$ and splitting the sum into two parts you get $(z-x)sum_{k=1}^{z-1} frac {x^{k}} {k!} +frac {zx^{z}} {z!}$ from which you cannot get rid of the sum.






            share|cite|improve this answer









            $endgroup$













            • $begingroup$
              Thank you @Kavi Rama Murthy, but you gave the answer. I just use the Gamma function for the last sum and I get the expression.
              $endgroup$
              – Y.L
              Jan 4 at 0:28














            0












            0








            0





            $begingroup$

            The answer is no. By making the change of avriable $k=z-n$ and splitting the sum into two parts you get $(z-x)sum_{k=1}^{z-1} frac {x^{k}} {k!} +frac {zx^{z}} {z!}$ from which you cannot get rid of the sum.






            share|cite|improve this answer









            $endgroup$



            The answer is no. By making the change of avriable $k=z-n$ and splitting the sum into two parts you get $(z-x)sum_{k=1}^{z-1} frac {x^{k}} {k!} +frac {zx^{z}} {z!}$ from which you cannot get rid of the sum.







            share|cite|improve this answer












            share|cite|improve this answer



            share|cite|improve this answer










            answered Jan 3 at 23:38









            Kavi Rama MurthyKavi Rama Murthy

            54.2k32055




            54.2k32055












            • $begingroup$
              Thank you @Kavi Rama Murthy, but you gave the answer. I just use the Gamma function for the last sum and I get the expression.
              $endgroup$
              – Y.L
              Jan 4 at 0:28


















            • $begingroup$
              Thank you @Kavi Rama Murthy, but you gave the answer. I just use the Gamma function for the last sum and I get the expression.
              $endgroup$
              – Y.L
              Jan 4 at 0:28
















            $begingroup$
            Thank you @Kavi Rama Murthy, but you gave the answer. I just use the Gamma function for the last sum and I get the expression.
            $endgroup$
            – Y.L
            Jan 4 at 0:28




            $begingroup$
            Thank you @Kavi Rama Murthy, but you gave the answer. I just use the Gamma function for the last sum and I get the expression.
            $endgroup$
            – Y.L
            Jan 4 at 0:28











            0












            $begingroup$

            Sticking to a more standard notation ($z$ normally denotes complex, not integral, number)



            begin{align}
            sum_{k=0}^n k frac{x^{n-k}}{(n-k)!}
            &= sum_{k=0}^{n} (n-k) frac{x^k}{k!}
            = nsum_{k=0}^n frac{x^k}{k!} - sum_{k=0}^n xfrac{d}{dx} frac{x^k}{k!}\
            &= nP_n(x) - xfrac{d}{dx}P_n(x)
            end{align}



            where $P_n(x) = sum_{k=0}^n frac{x^k}{k!}$ is the partial sum of the exponential's Taylor series. But as can be seen here, the incomplete gamma function $Gamma(n,x) = int_x^{infty} x^{n-1}e^{-x};dx$ may be expressed as



            $$Gamma(n,x) = (n-1)! e^{-x} sum_{k=0}^{n-1}frac{x^k}{k!}$$
            hence,
            $$P_n(x) = frac{Gamma(n+1,x)}{n!} e^x$$
            We may then compute



            begin{align}
            frac{d}{dx}P_n(x)
            &= P_n(x) frac{d}{dx} ln P_n(x) = P_n(x) left[,frac{frac{partial}{partial{x}}Gamma(n+1,x)}{Gamma(n+1,x)} + 1,right]\
            &= frac{Gamma(n+1,x)}{n!} e^x left[,frac{-x^ne^{-x}}{Gamma(n+1,x)} + 1,right]\
            &= frac{Gamma(n+1,x)}{n!} e^x - frac{x^n}{n!}
            end{align}



            Solving and simplifying, we get



            begin{align}
            sum_{k=0}^n k frac{x^{n-k}}{(n-k)!}
            &= nP_n(x) - xfrac{d}{dx}P_n(x)\
            &= frac{1}{n!} left[,(n-x)Gamma(n+1,x)e^x + x^{n+1},right]
            end{align}






            share|cite|improve this answer









            $endgroup$


















              0












              $begingroup$

              Sticking to a more standard notation ($z$ normally denotes complex, not integral, number)



              begin{align}
              sum_{k=0}^n k frac{x^{n-k}}{(n-k)!}
              &= sum_{k=0}^{n} (n-k) frac{x^k}{k!}
              = nsum_{k=0}^n frac{x^k}{k!} - sum_{k=0}^n xfrac{d}{dx} frac{x^k}{k!}\
              &= nP_n(x) - xfrac{d}{dx}P_n(x)
              end{align}



              where $P_n(x) = sum_{k=0}^n frac{x^k}{k!}$ is the partial sum of the exponential's Taylor series. But as can be seen here, the incomplete gamma function $Gamma(n,x) = int_x^{infty} x^{n-1}e^{-x};dx$ may be expressed as



              $$Gamma(n,x) = (n-1)! e^{-x} sum_{k=0}^{n-1}frac{x^k}{k!}$$
              hence,
              $$P_n(x) = frac{Gamma(n+1,x)}{n!} e^x$$
              We may then compute



              begin{align}
              frac{d}{dx}P_n(x)
              &= P_n(x) frac{d}{dx} ln P_n(x) = P_n(x) left[,frac{frac{partial}{partial{x}}Gamma(n+1,x)}{Gamma(n+1,x)} + 1,right]\
              &= frac{Gamma(n+1,x)}{n!} e^x left[,frac{-x^ne^{-x}}{Gamma(n+1,x)} + 1,right]\
              &= frac{Gamma(n+1,x)}{n!} e^x - frac{x^n}{n!}
              end{align}



              Solving and simplifying, we get



              begin{align}
              sum_{k=0}^n k frac{x^{n-k}}{(n-k)!}
              &= nP_n(x) - xfrac{d}{dx}P_n(x)\
              &= frac{1}{n!} left[,(n-x)Gamma(n+1,x)e^x + x^{n+1},right]
              end{align}






              share|cite|improve this answer









              $endgroup$
















                0












                0








                0





                $begingroup$

                Sticking to a more standard notation ($z$ normally denotes complex, not integral, number)



                begin{align}
                sum_{k=0}^n k frac{x^{n-k}}{(n-k)!}
                &= sum_{k=0}^{n} (n-k) frac{x^k}{k!}
                = nsum_{k=0}^n frac{x^k}{k!} - sum_{k=0}^n xfrac{d}{dx} frac{x^k}{k!}\
                &= nP_n(x) - xfrac{d}{dx}P_n(x)
                end{align}



                where $P_n(x) = sum_{k=0}^n frac{x^k}{k!}$ is the partial sum of the exponential's Taylor series. But as can be seen here, the incomplete gamma function $Gamma(n,x) = int_x^{infty} x^{n-1}e^{-x};dx$ may be expressed as



                $$Gamma(n,x) = (n-1)! e^{-x} sum_{k=0}^{n-1}frac{x^k}{k!}$$
                hence,
                $$P_n(x) = frac{Gamma(n+1,x)}{n!} e^x$$
                We may then compute



                begin{align}
                frac{d}{dx}P_n(x)
                &= P_n(x) frac{d}{dx} ln P_n(x) = P_n(x) left[,frac{frac{partial}{partial{x}}Gamma(n+1,x)}{Gamma(n+1,x)} + 1,right]\
                &= frac{Gamma(n+1,x)}{n!} e^x left[,frac{-x^ne^{-x}}{Gamma(n+1,x)} + 1,right]\
                &= frac{Gamma(n+1,x)}{n!} e^x - frac{x^n}{n!}
                end{align}



                Solving and simplifying, we get



                begin{align}
                sum_{k=0}^n k frac{x^{n-k}}{(n-k)!}
                &= nP_n(x) - xfrac{d}{dx}P_n(x)\
                &= frac{1}{n!} left[,(n-x)Gamma(n+1,x)e^x + x^{n+1},right]
                end{align}






                share|cite|improve this answer









                $endgroup$



                Sticking to a more standard notation ($z$ normally denotes complex, not integral, number)



                begin{align}
                sum_{k=0}^n k frac{x^{n-k}}{(n-k)!}
                &= sum_{k=0}^{n} (n-k) frac{x^k}{k!}
                = nsum_{k=0}^n frac{x^k}{k!} - sum_{k=0}^n xfrac{d}{dx} frac{x^k}{k!}\
                &= nP_n(x) - xfrac{d}{dx}P_n(x)
                end{align}



                where $P_n(x) = sum_{k=0}^n frac{x^k}{k!}$ is the partial sum of the exponential's Taylor series. But as can be seen here, the incomplete gamma function $Gamma(n,x) = int_x^{infty} x^{n-1}e^{-x};dx$ may be expressed as



                $$Gamma(n,x) = (n-1)! e^{-x} sum_{k=0}^{n-1}frac{x^k}{k!}$$
                hence,
                $$P_n(x) = frac{Gamma(n+1,x)}{n!} e^x$$
                We may then compute



                begin{align}
                frac{d}{dx}P_n(x)
                &= P_n(x) frac{d}{dx} ln P_n(x) = P_n(x) left[,frac{frac{partial}{partial{x}}Gamma(n+1,x)}{Gamma(n+1,x)} + 1,right]\
                &= frac{Gamma(n+1,x)}{n!} e^x left[,frac{-x^ne^{-x}}{Gamma(n+1,x)} + 1,right]\
                &= frac{Gamma(n+1,x)}{n!} e^x - frac{x^n}{n!}
                end{align}



                Solving and simplifying, we get



                begin{align}
                sum_{k=0}^n k frac{x^{n-k}}{(n-k)!}
                &= nP_n(x) - xfrac{d}{dx}P_n(x)\
                &= frac{1}{n!} left[,(n-x)Gamma(n+1,x)e^x + x^{n+1},right]
                end{align}







                share|cite|improve this answer












                share|cite|improve this answer



                share|cite|improve this answer










                answered Jan 4 at 1:18









                adfriedmanadfriedman

                3,161169




                3,161169






























                    draft saved

                    draft discarded




















































                    Thanks for contributing an answer to Mathematics Stack Exchange!


                    • Please be sure to answer the question. Provide details and share your research!

                    But avoid



                    • Asking for help, clarification, or responding to other answers.

                    • Making statements based on opinion; back them up with references or personal experience.


                    Use MathJax to format equations. MathJax reference.


                    To learn more, see our tips on writing great answers.




                    draft saved


                    draft discarded














                    StackExchange.ready(
                    function () {
                    StackExchange.openid.initPostLogin('.new-post-login', 'https%3a%2f%2fmath.stackexchange.com%2fquestions%2f3061092%2fan-equal-expression-to-sumz-n-0n-fracxz-nz-n%23new-answer', 'question_page');
                    }
                    );

                    Post as a guest















                    Required, but never shown





















































                    Required, but never shown














                    Required, but never shown












                    Required, but never shown







                    Required, but never shown

































                    Required, but never shown














                    Required, but never shown












                    Required, but never shown







                    Required, but never shown







                    Popular posts from this blog

                    Can a sorcerer learn a 5th-level spell early by creating spell slots using the Font of Magic feature?

                    Does disintegrating a polymorphed enemy still kill it after the 2018 errata?

                    A Topological Invariant for $pi_3(U(n))$